Can someone plz help..

Can Someone Plz Help..

Answers

Answer 1

Answer:

x = 40

3x=120

x+40 = 120

Step-by-step explanation:

The angles are vertical angles which means they are equal

3x= x+80

Subtract x from each side

3x-x =x+80-x

2x = 80

Divide by 2

2x/2 = 80/2

x = 40

3x = 3(40) =120

x+80 = 40+80 = 120


Related Questions

PLEASE HELP!!!

WILL MARK BRAINLIEST!!!


If A=28, then the measure of arc BA is_____

Answers

Answer:

124

Step-by-step explanation:

Have a nice day :)

Arc AC is 180º, arc BC is 56º:

[tex]180+56 +\arc{BA} = 360\implies BA = 124[/tex]

Write image as a complex number. Question 6 options: A) 3 + i B) 3 – i C) –3 + i D) –3 – i

Answers

Answer:

that's right B) 3 - √3i

Step-by-step explanation:

(12 - √-48) / 4 =

12/4 - √-48 /4

= 3 - √(-48/16)

= 3 - √-3

= 3 - √ 3i

[tex]\\ \rm\Rrightarrow \dfrac{12-\sqrt{-48}}{4}[/tex]

[tex]\\ \rm\Rrightarrow \dfrac{12}{4}-\dfrac{\sqrt{48}i}{4}[/tex]

[tex]\\ \rm\Rrightarrow 3-4\sqrt{3}i/4[/tex]

[tex]\\ \rm\Rrightarrow 3-\sqrt{3}i[/tex]

Which shows a graph of a linear equation in standard form Ax + By = C, where A = 0, B is positive, and C is negative? A coordinate plane with a vertical line passing through (1, negative 4), (1, 0) and (1, 4). A coordinate plane with a vertical line passing through (negative 2, negative 4), (negative 2, 0) and (negative 2, 4). A coordinate plane with a horizontal line passing through (negative 4, 4), (0, 4) and (negative 4, 4). A coordinate plane with a horizontal line passing through (negative 4, negative 3), (0, negative 3) and (4, negative 3).

Answers

Answer:

A coordinate plane with a horizontal line passing through (negative 4, negative 3), (0, negative 3) and (4, negative 3).

Step-by-step explanation:

Ax + By = C, where A = 0, B is positive, and C is negative

By= C or y= C/B which is negative

Since A is zero and C is negative, the line is parallel to x- axis and has negative value for y, so it is a horizontal line below zero

---------------

A coordinate plane with a vertical line passing through (1, negative 4), (1, 0) and (1, 4).

no, the line is horizontal

A coordinate plane with a vertical line passing through (negative 2, negative 4), (negative 2, 0) and (negative 2, 4).

no, the line is horizontal

A coordinate plane with a horizontal line passing through (negative 4, 4), (0, 4) and (negative 4, 4).

no, the y-values should be negative

A coordinate plane with a horizontal line passing through (negative 4, negative 3), (0, negative 3) and (4, negative 3).

yes, this is horizontal line below zero

Answer:

D

Step-by-step explanation:

How to solve and what is the answer for this question

Answers

Answer:

O DC = 65 and DE = 33.5

Step-by-step explanation:

DC is already given by AB

so DC is 65

DE would be BD split

DE is 33.5

Answer: O DC = 65 and DE = 33.5

Helpppppppppppppppp please y’all

Answers

[tex]\sf{3. \ \ Given : \dfrac{2}{3} - \dfrac{5}{4} + \dfrac{7}{2}}[/tex]

Let us solve the first two fractions : The LCM of 3 and 4 is 12

[tex]\sf{\implies \dfrac{4(2) - 5(3)}{12} + \dfrac{7}{2}}[/tex]

[tex]\sf{\implies \dfrac{8 - 15}{12} + \dfrac{7}{2}}[/tex]

[tex]\sf{\implies \dfrac{-7}{12} + \dfrac{7}{2}}[/tex]

LCM of 2 and 12 is 12

[tex]\sf{\implies \dfrac{-7 + 7(6) }{12}}[/tex]

[tex]\sf{\implies \dfrac{-7 + 42}{12}}[/tex]

[tex]\sf{\implies \dfrac{35}{12}}[/tex]

[tex]\sf{\implies 2 +\dfrac{11}{12}}[/tex]

[tex]\sf{\implies 2 \dfrac{11}{12}}[/tex]

---------------------------------------------------------------------------------------

[tex]\sf{4. \ \ Given : \dfrac{(2 - 7)^2 + 5}{3}}[/tex]

[tex]\sf{\implies \dfrac{(-5)^2 + 5}{3}}[/tex]

[tex]\sf{\implies \dfrac{25 + 5}{3}}[/tex]

[tex]\sf{\implies \dfrac{30}{3}}[/tex]

[tex]\sf{\implies 10}[/tex]

Answer:

2/3-5/4+7/2

Step-by-step explanation:

collect like terms. that is the positive and negative terms2/3+7/2-5/4lcm of 3 and 2 is 6 hence 2/3+7/2= 25/625/6-5/4....lcm of 6 and 4 is 12 hence 25/6-5/4=35/12 the answer in its mixed fraction is 2 and 11/12

Item 23
What kind of angle is angle BCA ?

acute

obtuse

right

Answers

Answer:

BCA is obtuse

Step-by-step explanation:

BCA = 98 degrees

greater than 0 to less than 90 is acute angles

90 - right angle

greater than 90 to less than 180 is obtuse

98 is obtuse

Which equation demonstrates the additive identity property?
(7 + 4 i) + (7 minus 4 i) = 14
(7 + 4 i) + 0 = 7 + 4 i
(7 + 4 i) (1) = 7 + 4 i
(7 + 4 i) + (negative 7 minus 4 i) = 0

Answers

Answer:

first you add then you subtract

Step-by-step explanation:

Answer:

The answer is (B) - (7+4 i) + (7 minus 4 i) = 14

Step-by-step explanation:

While watching the Kentucky Derby horse races, you decide to guess who will win 1st, 2nd, and 3rd place. There are 17 horses in the race. What is the probability you choose all three places correctly

Answers

Answer:

1/17

Step-by-step explanation:

I he had even got 1 out of the 3 places the probability would be 3/17

But as he is guessing all three places it becomes 1/17

The probability of getting the 1st,2nd, and 3rd position will be 1/17.

What is the probability?

Probability is defined as the ratio of the number of favorable outcomes to the total number of outcomes in other words the probability is the number that shows the happening of the event.

Given that while watching the Kentucky Derby horse races, you decide to guess who will win 1st, 2nd, and 3rd place. There are 17 horses in the race.

The probability is calculated by the formula,

Probability = Number of favorable outcomes / Number of sample

Number of favorable outcomes = 1

Number of samples = 17

Probability = Number of favorable outcomes / Number of sample

Probability = 1/17

Therefore, the probability of getting the 1st,2nd, and 3rd position will be 1/17.

To know more about probability follow

https://brainly.com/question/24756209

#SPJ5

PLEASE HELP!!!

WILL MARK BRAINLIEST!!!

If AC= 12cm, what is the length of arc AB to the nearest whole number?

Thank you! Multiple choice!

Answers

Answer:

45

Step-by-step explanation:

Have a nice day :)

Type the correct answer in each box.

Find the solution for this system of equations.
2x + 4y = 8
x = 3y − 6

Answers

Answer X=0
AnswerY=2

Answer:

X=0

Y=2

Step-by-step explanation:

Just finished and i got it right

The linear function g is defined by g(x) = 4x.

a. Determine g(3).


b. Determine g(2).


c. Determine g(3) - g(2)

Answers

Answer:

the answer is D.

Step-by-step explanation:

well, the y-intercept for f(x) is at  

as you can see from the slope-intercept form is at 4, and it has an slope of 2/3.

for g(x), well an y-intercept is when x = 0, what is it from that table?  well, is at 0,3, so when x = 0, y = 3, so no dice on that one.

c)

whenever an x-intercept occurs, y = 0, for f(x) that's at  

what about the x-intercept for g(x)?  well, let's check, when is y = 0?  aha!  at -9, 0, so when y = 0, x = -9, so no dice on that one either.

d)

well, what is the slope of g(x) anyway?  well, let's pick two points off the table to get it hmmm the first two let's use,

and from a), using the slope-intercept form, we know f(x) has a slope of 2/3.

well, 2/3 is larger than 1/3, so no dice.

b)

well, you already know.

Correct answer to your question is letter D

Patterson Planning Corp.,

You have been hired by Patterson Planning Corp., an events planning company that recently had a fire in which some of the accounting records were damaged.

In reviewing the fixed asset records, you find three depreciation schedules that are not labeled. They are listed in the following table. One of the assets has a depreciation rate of $4.30 per hour.

Answers

Answer:

Total depreciation now will be $51500 and the depreciation for you will be $5575

Step-by-step explanation:

Your welcome! :)

Which statement is true about the function shown in the graph?

Answers

Answer:

A

Step-by-step explanation:

The function that is  shown in the graph is strictly decreasing

What is a function?

"A function from a set X to a set Y assigns to each element of X exactly one element of Y. The set X is called the domain of the function and the set Y is called the codomain of the function."

The given function has a value of (- 5) at a certain point.

After that the function is decreased to a value of (- 10).

Therefore, we can conclude that the function is strictly decreasing.

Learn more about a function here:

https://brainly.com/question/11943224

#SPJ2

6. Which of the following is equivalent to the expression shown?
(x - 5)(x + 5)
(A) x2 – 10x - 25
(C) x2 – 25
x2 - 10x + 25
(D) x2 + 25
(x–52) (x+5)
x²-524
X2-10
I’m stuck on this help anyone ? :(

Answers

Answer:

its x^2 -25

Step-by-step explanation:

simply we know,

a^2-b^2=(a-b)(a+b)

A file that is 226 megabytes is being downloaded if the download is 12.&% complete, how many megabytes have been downloaded? Round your answer to the nearest tenth

Answers

Answer:

28.7

Step-by-step explanation:

12.7% = 0.127

226 * 0.127 = 28.702

Rounded

28.7

Multiply x^2+2x+3 times 4x^2-2x+1

Answers

Answer:

4x^4 + 6x^3 + 9x^2 − 4x + 3

Step-by-step explanation:

Simplify the expression. 4x^4 + 6x^3 + 9x^2 − 4x + 3

Step-by-step explanation:

Answer:

answer for this solution is 4x^4+6x^3+9x^2-4x+3

What is the distance between (-5, -6) and (-3, -8)?

Answers

I think it’s that 2√2

Please help, im stuck. ​

Answers

Answer:

4x^2 + 12x + 9

Step-by-step explanation:

(2x + 3)^2

it is in the form of (a + b)^2

(a + b)^2 = a^2 + 2*a*b + b^2

(2x + 3)^2

(2x)^2 + 2*2x*3 + (3)^2

4x^2 + 12x + 9

In the diagram of circle O shown to the right, PA and PB are tangent to circle O at points A and B, respectively. If mACB  266, then mAPB 

Answers

Answer: 86 degrees

Step-by-step explanation:

write a quadratic function f whose zeros are 2 and -6

Answers

Answer:

hope this will help u

Step-by-step explanation:

⇒  Given zeros are α=2 and β=−6.

⇒  Sum of zeros =α+β=2+(−6)=−4

⇒  Product of zeros =α×β=2×(−6)=−12

⇒  Quadratic polynomial =x  

2

−(α+β)x+(α×β)

⇒  Quadratic polynomial =x  

2

−(−4)x+(−12)

∴   Quadratic polynomial =x  

2

+4x−12

The quadratic function having 2 and -6 as zeros is f(x) = x² + 4x - 12.

What is the quadratic function with roots 2 and -6?

Given the parameters:
Zeros of the function: 2 and -6.

To find the quadratic function f(x) with zeros at 2 and -6, first set up the equation:

x = 2 and x = -6

These are the real solutions to the quadratic equation:

This means that; ( x - 2 ) and ( x + 6 ) are the factors of the quadratic equation:

Hence:

( x - 2 )( x + 6 ) = 0

Expand using distributive property:

x( x + 6 ) - 2(x + 6 )  = 0

x² + 6x - 2x - 12 = 0

Collect and add like terms:

x² + 4x - 12 = 0

Therefore, the function will be:
f(x) = x² + 4x - 12.

The quadratic function is f(x) = x² + 4x - 12.

Learn more about quadratic equations here: brainly.com/question/1863222

#SPJ6

Analyze the diagram below and complete the instructions that follow.
Find a, b, and c.

Answers

Answer:

Step-by-step explanation:

for side a

take 60 degree as reference angle

using tan rule

tan 60 =opposite /adjacent

[tex]\sqrt{3}[/tex] = a/6

do cross multiplication

6*[tex]\sqrt{3}[/tex] =a*1

[tex]6\sqrt{3} = a[/tex]

for b

using pythagoras theorem

a^2 + b^2 =c^2 (here a nd b are the legs of a right triangle and c is hypotenuse)

6^2 + [tex](6\sqrt{3})^2 = b^2[/tex]

36 + 6*6*3 = b^2

36 + 108 = b^2

144 = b^2

[tex]\sqrt{144}[/tex] = b

14 = b

for angle c

take 45 degree as reference angle

using cos rule

cos 45 = adjacent/hypotenuse

[tex]\frac{1}{\sqrt{2} } = \frac{c}{12}[/tex]

do cross multoplication

[tex]c*\sqrt{2} = 12*1[/tex]

[tex]c = \frac{12}{\sqrt{2} } \\\\c = 6\sqrt{2[/tex]

brainlest! to first one pls help mehh

Answers

Answer:

A

Step-by-step explanation:

Answer:

option a and b

Step-by-step explanation:

peanuts=1/4 cup    ratio=1:4

raisins=1/2 cup        ratio=1:2

chocolate chips=3/4 cup      ration=3:4

granola =1 cup            ration=1:1

a. ratio of cop of rasins to cup of peanuts=2:1 or 2/1

1/2 = 1/4=2/1

do cross multiplication

4*1=2*1=2/1

4/2=2/

2/1=2/1

2:1=2:1  this is satisfied

b.ratio of cup of granola to cup of peanuts=1:4 or 1/4

1/1=1/4

do crss multiplication

1*1/4*1=1/4

1/4=1/4

1:4=1:4      this is satisfied as both are equal

c.the ratio of cup of chocolate chips to cup of raisins=3:1 or 3/1

3/4=1/2

do cross multiplication

3*2=4*1=3/1

6/4=3/1

6:4=3:1   this is not satisfied as they are different.

d.the ratio of cup of raisins to cup of granola=1:3 or 1/3

1/2=1/1

do cross multiplication

1*1=2*1=1/3

1/2=1/3

1:2=1:3    this is not satisfied as they are different.

e. the ratio of cup of granola to cup of chocolate chips =4:3 or 4/3

1/1=3/4

do cross multiplication

3*1=4*1=4/3

3/4=4/3

3:4=4:3    this is not satisfied as they are not equal

therefore ur answer is option a and b.

Suppose you deposit $3000 in a savings account that pays interest at an annual rate of 4%.
What is the growth factor?

Answers

Answer:

$750 for 4 percent annually

Step-by-step explanation:

3000/4 =750 But it also depends on if you want that 4 percent to grow.

The pentagon shown could be reflected over which line to be carried onto itself?

Answers

Answer:

on the x axis because it is reflected there

Plz help solve this thanks

Answers

Answer:

y= -3/2

Step-by-step explanation:

........................

Answer to the question

Answers

Answer:

7

Step-by-step explanation:

Step-by-step explanation:  

AB//CD (Opposite sides of parallelogram are parallel)  

Angle A + Angle B = 180 (Sum of interior angles=180)  

179 + 3x - 20 = 180  

3x - 20 = 180 - 179

3x - 20 = 1

3x = 1 + 20

3x = 21

x = 21/3  

x = 7


how to find coefficient of x²in 2+x²+c

Answers

Answer:

1 is cofficient of X ²

I hope it's helpful for u ...

Solve the system of equations.
y=8x-6
y=5x+9​

Answers

Answer:

x= 5 , y = 34

Step-by-step explanation:

8x-6=5x+9

3x-6=9

3x=15

x=5

-

8(x5) - 6 =

34

25(x5) +9 =

34

Answer:

x = 5 and y = 34

Step-by-step explanation:

I used substitution method and did this ;

8x - 6 = 5x + 9 .....put like terms together

8x - 5x = 9 + 6

3x = 15

x = 5

y = 8 (5) - 6

y = 40 - 6

y = 34

Joelle would like to tip her hairdresser 20%. If her haircut costs $38.00, which expression should she use to find the tip amount?

Answers

20%=1/5 so the tip amount is 1/5*($38.00)=$7.60

Amber is solving the inequality X+61- 12 < 13 by graphing. Which equations should Amber graph?
O Y1 = +6 Y2 - 25
O y1= x+6y2 = 25
O Y1 = X+61.72 = 13
O Y1 = X+6.82 = 13

Answers

Therefore, Amber should graph the equation Y1 = X+61.

What is inequality?

An inequality is a mathematical statement that compares two values or expressions using one of the inequality symbols: < (less than), > (greater than), ≤ (less than or equal to), ≥ (greater than or equal to), or ≠ (not equal to). Inequalities are used to describe a range of possible values for a variable, rather than a single value. Inequalities can be solved by performing operations on both sides of the inequality symbol, just like equations. However, there are some important differences between solving equations and solving inequalities. For example, if we multiply or divide both sides of an inequality by a negative number, we need to reverse the direction of the inequality symbol.

Here,

To graph the inequality X+61- 12 < 13, we need to rearrange it to the form Y < MX + B, where Y is on the left-hand side and X is on the right-hand side. This gives:

Y < -X + 64

Now we can graph the line Y = -X + 64 and shade the region below it, since we want Y to be less than this expression.

Out of the equations given, Y1 = X+61 is the closest to the inequality we are trying to graph. However, the coefficients of X and Y are reversed, so we need to rearrange it to the desired form:

Y - X < 61

Y < X + 61

To know more about inequality,

https://brainly.com/question/30239204

#SPJ1

Other Questions
Guided PracticeIdentify the independent clause in this sentence.The tiger population has increased over the years since it has been protected by new conservation laws.A.The tiger population has increased over the yearsB.since it has been protected by new conservation laws How is the information in the first passage similar to the information in the second passage? A. Both passages explain that herbs are leafy green plants. B. Both passages explain the history of spices, and spice trade. C. Both passages mention Sanskrit Ramayana and Genesis. D. Both passages mention per capita use of spice in the West. What is one of the four freedoms that Roosevelt identifies in his FourFreedoms speech?A. Freedom from wantB. Freedom to bear armsC. Freedom from tyrannyD. Freedom of education Which is a factor of 10?OA. 40B. 10C. 30D. 20 A business had a profit of $42,000 in 1994.Then its profit decreased by 8% cach ycarfor 6 years. How much did the businessearn in 2000? Martin deposits $200 in a savings account that earns 5% annual interest. Four years later, Cary deposits $200 in an account earning the same interest. Let M represent the balance in Martins account and let C represent the amount of money in Carys account. Which of the equations below represents a line perpendicular to the x-axis?A. X=3B. X=3yC. X=yD. X=-3y Dave's Duds reported cost of goods sold of $1,600,000 this year. The inventory account increased by $130,000 during the year to an ending balance of $465,000. What was the cost of merchandise that Dave's purchased during the year? 7. The government of Mexico declared blue sharks an endangered species. They put tags on 36 blue sharks and released them. Later, they corral 130 blue sharks; among those blue sharks, 20 were tagged. Find the best estimate for the blue shark population? What is likely to happen to a healthy population that is experiencing exponential growth ? which department would you approach if your human rights had been violated? What is the awnsers helppppp Question 8 of 10What question might a reader ask to determine whether a narrator isunreliable?A. Does the narrator feel emotions toward certain people?B. How often does the narrator mention other characters?C. Does the narrator seem interested in specific details?D. How honestly does the narrator represent the world? How do I solve this math problem? The answer is: x = a^2/a-3 slecet the correct answer what does rule of conduct mean? Find the volume of the prism or pyramid below IS THIS CORRECT IF NOT WHATS THE ANSWER FAST PLS A person riding a bike has a total mass of 95 kg, if the person bike system is moving at a velocity of 5 m/s, how much kinetic energy is there? Short ResponseShantelle wants to rent a mid-size car for one day. Value Rent-a-Car charges $20 a day plus $0.20 permile, and Best Rent-a-Car charges $30 a day plus $0.10 per mile for rental of the same mid-size car.Part A) Write an equation in slope intercept form, y=mx+b, for each car:Value Rent-a-Car: y=0.20x+20Best Rent-a-Car: y=0.10x+30Part B: Determine the mileage where the rental charges are the same.